Trostruki integral po figuri

PostPoslato: Utorak, 24. Januar 2017, 17:38
od Littlefinger
Postavio sam ovo pitanje na proslu temu sto sam napravio u dijelu grafik funkcija, ali sam shvatio da joj tamo nije mjesto pa sam odlucio da je ovdje postavim. Kako nijesam stigao da izbrisem taj post, ako nekom nije problem neka ga izbrise da ne pravi guzvu.

treba da izracunam sledeci integral:
[dispmath]\iiint\left(x^2+y^2\right)\,\mathrm dx\,\mathrm dy\,\mathrm dz[/dispmath] za oblast [inlmath]x^2+y^2+z^2=a^2[/inlmath] gdje je [inlmath]z>0[/inlmath] tj. samo gornja polulopta.

Rjesenje ispada [inlmath]\frac{4\cdot a^5\cdot\pi}{15}[/inlmath]

Koristio sam sferne koordinate i dobijam tacan rezultat kada postavim granice za ugao [inlmath]0<\theta<\frac{\pi}{2}[/inlmath] i [inlmath]0<\phi<\pi[/inlmath] tj. prepolovim krug, a zatim to sto sam dobio pomnozim sa [inlmath]2[/inlmath]. Ali ako stavim granicu [inlmath]0<\theta<\pi[/inlmath] citav integral sto dobijem koji zavisi od sinusa se izjednacava sa nulom i dobijem drugaciji rezultat.

Poslednja formula koju dobijem izgleda ovako:
[dispmath]\int\mathrm d\phi\cdot\left(-\sin^2\theta\cdot\cos\theta+\frac{2\cdot\sin^3\theta}{3}\right)\cdot\frac{a^5}{5}[/dispmath] gdje je [inlmath]0<\theta<\frac{\pi}{2}[/inlmath] i [inlmath]0<\phi<\pi[/inlmath] za prvi nacin i [inlmath]0<\theta<\pi[/inlmath] i [inlmath]0<\phi<2\cdot\pi[/inlmath] za drugi nacin

Vazno je napomenut da sam ja koristio drugacije sferne koordintate od onih na vasem sajtu tj.
[dispmath]z=r\cdot\cos\theta;\quad x=r\cdot\cos\phi\cdot\sin\theta;\quad y=r\cdot\sin\phi\cdot\sin\theta[/dispmath] Vjerujem da je problem do ugla [inlmath]\theta[/inlmath] i da postoje neka ogranicenja dokle taj ugao moze ici, da li to sto je sinus pozitivna funkcija na intervalu [inlmath]0<\theta<\pi[/inlmath] ima takav uticaj da kada stavimo te granice na suprotne krajeve integrala da se oni pokrate?

Re: Trostruki integral po figuri

PostPoslato: Četvrtak, 26. Januar 2017, 07:39
od Daniel
Littlefinger je napisao:Koristio sam sferne koordinate i dobijam tacan rezultat kada postavim granice za ugao [inlmath]0<\theta<\frac{\pi}{2}[/inlmath] i [inlmath]0<\phi<\pi[/inlmath] tj. prepolovim krug, a zatim to sto sam dobio pomnozim sa [inlmath]2[/inlmath].

Znači, posmatrao si samo onu polovinu date polusfere (dakle, četvrtinu sfere) u kojoj važi da je, posmatrano u Dekartovim koordinatama, [inlmath]y[/inlmath] pozitivno, pa zatim rezultat množio sa [inlmath]2[/inlmath]. To je opravdano, budući da će data funkcija [inlmath]x^2+y^2[/inlmath] zbog ovog kvadriranog [inlmath]y[/inlmath], imati simetrično raspoređene vrednosti za [inlmath]y<0[/inlmath] i za [inlmath]y>0[/inlmath].

Littlefinger je napisao:Ali ako stavim granicu [inlmath]0<\theta<\pi[/inlmath] citav integral sto dobijem koji zavisi od sinusa se izjednacava sa nulom i dobijem drugaciji rezultat.

Ali, zašto [inlmath]\theta[/inlmath]? To je ugao koji računaš od pozitivnog dela [inlmath]z[/inlmath]-ose do prave koja spaja posmatranu tačku s koordinatnim početkom. Ako je rečeno da se posmatra ona polovina sfere u kojoj je [inlmath]z>0[/inlmath], tada [inlmath]\theta[/inlmath] mora ići od [inlmath]0[/inlmath] do [inlmath]\frac{\pi}{2}[/inlmath], jer je [inlmath]z=r\cos\theta[/inlmath], što znači da [inlmath]\cos\theta[/inlmath] mora biti pozitivan. Ako bi išao preko [inlmath]\frac{\pi}{2}[/inlmath], zašao bi u onu drugu polovinu sfere u kojoj je [inlmath]z<0[/inlmath] (a koja te po uslovu zadatka ne interesuje).
Umesto toga, potrebno je da ugao [inlmath]\phi[/inlmath], koji si u prvom načinu terao od [inlmath]0[/inlmath] do [inlmath]\pi[/inlmath] pa zatim rezultat množio sa [inlmath]2[/inlmath], sada teraš od [inlmath]0[/inlmath] do [inlmath]2\pi[/inlmath], i time opišeš pun krug oko [inlmath]z[/inlmath]-ose.

Littlefinger je napisao:Poslednja formula koju dobijem izgleda ovako:
[dispmath]\int\mathrm d\phi\cdot\left(-\sin^2\theta\cdot\cos\theta+\frac{2\cdot\sin^3\theta}{3}\right)\cdot\frac{a^5}{5}[/dispmath] gdje je [inlmath]0<\theta<\frac{\pi}{2}[/inlmath] i [inlmath]0<\phi<\pi[/inlmath] za prvi nacin i [inlmath]0<\theta<\pi[/inlmath] i [inlmath]0<\phi<2\cdot\pi[/inlmath] za drugi nacin

Prvo, nije mi jasno kako si dobio izraz unutar zagrade. Da li si krenuo od sledećeg trojnog integrala?
[dispmath]\int\limits_0^\pi\,\mathrm d\phi\int\limits_0^{\pi/2}\sin^3\theta\,\mathrm d\theta\int\limits_0^ar^4\,\mathrm dr[/dispmath] Drugo, ovako kako si napisao, ispada da si u drugom načinu za oba ugla, i za [inlmath]\theta[/inlmath] i za [inlmath]\phi[/inlmath], udvostručio opseg u odnosu na prvi način. Kao što rekoh, potrebno je to da učiniš samo za ugao [inlmath]\phi[/inlmath], koji bi sada išao od [inlmath]0[/inlmath] do [inlmath]2\pi[/inlmath], dok [inlmath]\theta[/inlmath] i dalje ide od [inlmath]0[/inlmath] do [inlmath]\frac{\pi}{2}[/inlmath].

Littlefinger je napisao:Vazno je napomenut da sam ja koristio drugacije sferne koordintate od onih na vasem sajtu tj.
[dispmath]z=r\cdot\cos\theta;\quad x=r\cdot\cos\phi\cdot\sin\theta;\quad y=r\cdot\sin\phi\cdot\sin\theta[/dispmath]

Ne znam na koji deo ovog sajta misliš.

Littlefinger je napisao:Vjerujem da je problem do ugla [inlmath]\theta[/inlmath] i da postoje neka ogranicenja dokle taj ugao moze ici, da li to sto je sinus pozitivna funkcija na intervalu [inlmath]0<\theta<\pi[/inlmath] ima takav uticaj da kada stavimo te granice na suprotne krajeve integrala da se oni pokrate?

Za ograničenja već odgovorih – dakle, uslov [inlmath]z>0[/inlmath] ograničava ugao [inlmath]\theta[/inlmath] da ide od [inlmath]0[/inlmath] do [inlmath]\frac{\pi}{2}[/inlmath].
A čak i ako bi, uprkos uslovu zadatka, terao [inlmath]\theta[/inlmath] od [inlmath]0[/inlmath] do [inlmath]\pi[/inlmath], opet ne bi trebalo da dobiješ nulu, al' ti je dobijaš zbog tog izraza unutar zagrade, za koji mislim da je pogrešan... Proveri ga još jednom.